0 Daumen
546 Aufrufe

$$ a)  z^{3} = j      $$
$$ b) z^{4} = 16 z^{2} - 100 $$


Bitte um Hilfe ich komme nicht drauf :(

Avatar von

2 Antworten

0 Daumen

z^3 = j   Bedenke   j = cos(90°) + j*sin(90°)

und für die 3. Wurzel musst du den Winkel durch 3 teilen

also 1. Lösung z= cos(30°) + j*sin(30°) = 0,5*√3 + j/2

Das gleiche mit 90°+360° und 90°+2*360°.


z^4 = 16z^2 - 100

<=>  z^4 - 16z^2 + 100 = 0

<=>  z^4 - 16z^2 + 64 =  -36

<=>  (z^2 -8) ^2 =  -36

<=>  z^2 -8 =  6j  oder  z^2 -8 =  -6j

<=>  z^2  =  8+6j  oder  z^2  =  8-6j

jetzt noch die Wurzel ausrechnen gibt die 4 Lösungen.

3+j   ;  -3-j  ;  3-j  und  -3+j .

Avatar von 288 k 🚀

warum macht man im dritten schritt die 36 rüber

Damit auf der linken Seite eine binomische Formel übrig bleibt.

0 Daumen

Aloha :)

$$z^3=i=\cos\frac{\pi}{2}+i\,\sin\frac{\pi}{2}=e^{i\pi/2}$$Jetzt die dritte Wurzel ziehen und alle 3 symmetrischen Lösungen berücksichtigen:$$z_k=e^{i\pi/6+\frac{2\pi k}{3}}\;\;;\;\;k=0,1,2$$$$z_0=e^{i\pi/6}=\cos\frac{\pi}{6}+i\sin\frac{\pi}{6}=\frac{\sqrt 3}{2}+\frac{i}{2}=\frac{\sqrt3+i}{2}$$$$z_1=e^{i5\pi/6}=\cos\frac{5\pi}{6}+i\sin\frac{5\pi}{6}=-\frac{\sqrt 3}{2}+\frac{i}{2}=\frac{-\sqrt3+i}{2}$$$$z_2=e^{i3\pi/2}=\cos\frac{3\pi}{2}+i\sin\frac{3\pi}{2}=-i$$


$$z^4=16z^2-100$$$$\Leftrightarrow\quad z^4-16z^2+100=0$$$$\Leftrightarrow\quad (z^2-6z+10)(z^2+6z+10)=0$$Jetzt die pq-Formel auf die erste und die zweite Klammer anwenden:$$z_{0;1}=3\pm\sqrt{9-10}=3\pm i$$$$z_{2;3}=-3\pm\sqrt{9-10}=-3\pm i$$

Avatar von 148 k 🚀

Ein anderes Problem?

Stell deine Frage

Willkommen bei der Mathelounge! Stell deine Frage einfach und kostenlos

x
Made by a lovely community